Você está na página 1de 56

Raid M.

Al-Ani

150 MCQs in
Otolaryngology
With Explanatory Answers
Preface
___________________________________________

According to my best knowledge, MCQs are an essential component for


the assessment of medical students in all medical institutions, and dealing
with them is a challenge.
The trick to pass any exam is to identify correct study material and to
practice them at the very beginning. I have tried my best to bring out this
book in a clear, simplified and explanatory manner. The key feature of this
book is the division of it into two parts, the first part is for questions in
Otology, Laryngology and Rhinology, and the other part is for answering
and explanation of these questions, so that students go through these
questions, as they finish theory lectures of the respective topic. I would
advise a similar approach to students in all subjects.
This book is the first work in Department of Surgery/ENT, Medical
College, University of Anbar. And I am very happy to receive any
suggestions to improve on the present book and will be a guide to improve
the subsequent works.

Raid M. Al-Ani

3
Contents
___________________________________________
Part 1 – Questions in Otolaryngology

1. Questions in Otology……………………………………………. 5
2. Questions in laryngology……………………………………….. 13
3. Questions in Rhinology…………………………………………. 21

Part 2 – The answers

1. Answers for questions in Otology……………………………… 29


2. Answers for questions in laryngology…………………………. 38
3. Answers for questions in Rhinology…………………………… 49

4
Single best response questions
Questions for Otology
1. Regarding cholesteatoma, which of the following is true?
A. It consists of squamous epithelium.
B. It is a malignant tumour.
C. It should be left untreated.
D. It may metastasise to distant sites.
E. It is mainly treated medically.
2. Tympanic membrane develops from:
A. Ectoderm.
B. Endoderm.
C. Mesoderm.
D. A, B and C.
E. A and C.
3. Stapes footplate covers:
A. Round window.
B. Oval window.
C. Sinus tympani.
D. Aditus ad antrum.
E. Choclear aqueduct.
4. The auricle attains 90-95% of adult size by:
A. 5-6 years.
B. Birth.
C. 9-12 years.
D. 11-15 years.
E. 13-18 years.
5. Communication between middle ear and Eustachian tube is obliterated
surgically in
A. Cortical mastoidectomy.
B. Radical mastoidectomy.
C. Myrigoplasty.
D. Modified radical mastoidectomy.
E. Bondyʼs mastoidectomy.
6. Treatment of choice for glue ear which presented for 6 months is
A. Conservative.
B. Adenoidectomy.
C. Myringotomy with ventilation tube insertion.
D. Myringotomy with cold knife.
E. Myringotomy with diode laser.
7. The cough response caused while cleaning the ear canal is mediated by
Stimulation of:
A. The 5th cranial nerve.
B. Innervation of external ear canal by C1 and C2.
C. Branches of the 7th cranial nerve.
D. The 8th cranial nerve.
E. The 10th cranial nerve.

5
8. Which of the following is a cause of sensorineural hearing loss:
A. Prespyacusis.
B. Tympanosclerosis.
C. Otosclerosis.
D. Longitudinal fracture of the temporal bone.
E. Cholesteatoma.
9. Which is the investigation of choice in assessing hearing loss in neonates?
A. Impedance audiometry.
B. Behavioral audiometry.
C. Free field audiometry.
D. Brain-stem evoked response audiometry.
E. Pure tone audiometry.
10. Use of Siegelʼs speculum during ear examination provides all except:
A. Magnification.
B. Visualization.
C. Assessment of movement of the tympanic membrane.
D. Removal of foreign body.
E. As applicator for the powdered antibiotic to ear.
11. All are true for Gradenigoʼs syndrome except
A. It is associated with conductive hearing loss.
B. It is seen in petrositis.
C. It leads to involvement of the 5th and 6th cranial nerves.
D. It is characterized by retro-orbital pain.
E. Otalgia.
12. In right middle ear pathology, Weberʼs test will be
A. Lateralized to right side.
B. Positive.
C. Lateralized to left side.
D. Centralized.
E. Normal.
13. Which of the following is not a typical feature of malignant otitis externa?
A. Mitotic figures are high.
B. Caused by Pseudomonas aeruginosa.
C. Patient are usually immune compromised.
D. Presence of granulation tissue.
E. Patients are usually old.
14. Otoacoustic emissions are produced by:
A. Inner hair cells.
B. Basilar membrane.
C. Auditory nerve.
D. Outer hair cells.
E. Utricle.

6
15. Speech frequencies include:
A. 500 1000 2000 Hz.
B. 125 250 2000 Hz.
C. 250 500 1000 Hz.
D. 1000 2000 3000Hz.
E. 2000 3000 4000Hz.
16. Decreased bone conduction in an audiogram indicates:
A. Glue ear.
B. Tympanic membrane perforation.
C. Damage to cochlea.
D. Ossicular dislocation.
E. Ossicular fixation.
17. Traumatic perforation differ from infective perforation of the ear drum in the:
A. Size of perforation.
B. Shape of perforation.
C. Number of perforations.
D. Site of perforation.
E. None of the above.
18. Caloric test determines function of:
A. Posterior semicircular canal.
B. Saccule.
C. Utricle.
D. Lateral semicircular canal.
E. Superior semicircular canal.
19. Treatment of dry traumatic rupture of tympanic membrane is:
A. Antibiotic ear drops.
B. Ear pack soaked with antibiotic.
C. Myringoplasty.
D. Protection of ear against water.
E. Systemic antibiotics.
20. All are true about ear wax except:
A. pH is acidic in normal healthy canals.
B. Contains a bactericidal enzyme.
C. Is a combination of secretions of sebaceous and apocrine glands.
D. Needs to be removed periodically.
E. Secreted from outer third of external auditory canal.
21. Type B tympanogram is found in:
A. Normal person.
B. Tympanosclerosis.
C. Otosclerosis.
D. Secretary otitis media.
E. Disconnection of the ossicles.

7
22. Which of the following is not an extracranial complication of chronic
suppurative otitis media:
A. Sigmoid sinus thrombosis.
B. Facial nerve palsy.
C. Squamous cell carcinoma.
D. Labyrinthitis.
E. Hearing loss.
23. Unilateral referred otalgia is due to all of the following except:
A. Peritonsillar abscess.
B. Ulcer oral tongue.
C. Allergic rhinitis.
D. Tempromandibular joint dysfunction.
E. Cancer of pyriform fossa.
24. Obliteration of postauricular sulcus is found in:-
A. Acute mastoiditis.
B. Acute petrositis.
C. Malignant otitis externa.
D. Otomycosis.
E. Gradingoʼs syndrome.
25. The augmentation power of the sound wave by the tympanic membrane is:-
A. 14 times.
B. 1.4 times.
C. 18 times.
D. 17 times.
E. 22 times.
26. Aim of mastoid surgery in CSOM which should receive first priority is:
A. Making the ear dry.
B. Rendering the ear safe.
C. Improvement in hearing.
D. Preservation of hearing.
E. Eradication of infection.
27. Most common cause for bilateral conductive deafness in a child is:
A. Otitis media with effusion.
B. Otosclerosis.
C. Acute otitis media.
D. Congenital cholesteatoma.
E. Chronic suppurative otitis media.
28. A child aged 3 years presented with severe sensorineural deafness, he was
prescribed hearing aids but showed no improvement. What is the next line of
management?
A. Fenestration.
B. Stapes mobilization.
C. Cochlear implant.
D. Conservative.
E. Mastoidectomy.

8
29. The most common cause of peripheral episodic vertigo is:
A. Benign paroxysmal positional vertigo.
B. Meniereʼs disease.
C. Acoustic neuroma.
D. Vascular occlusion of labyrinthine artery.
E. Labyrinthitis.
30. A fifty year old male diabetic patient has developed itching in his ears. On
otoscopy there is debris with black specks in external auditory canal. The
treatment of choice in this case is:-
A. Otosporin ear drops.
B. Gentamicin ear drops.
C. Salicylic acid in spirit drops.
D. Repeated suction toilet with antifungal drops.
E. Oral anti-fungal drugs.
31. Adenoidectomy is indicated in all of the following conditions except:
A. Otitis media with effusion.
B. Nasal obstruction due to adenoidal hyperplasia.
C. Allergic rhinitis in children.
D. Recurrent otitis media in children.
E. Sleep apnea syndrome.
32. Complications of mumps include all except:
A. Unilateral sensorineural hearing loss.
B. Pancreatitis.
C. Palatal paralysis.
D. Orchitis.
E. Bilateral sensorineural hearing loss.
33. External auditory canal extends from …… to tympanic membrane.
A. Attic.
B. Lobule.
C. Isthmus.
D. Concha.
E. Umbo.
34. All of the following regarding pure tone audiometry is true except:
A. Is an objective test.
B. It uses specific tones to give place specific responses.
C. Is used to determine the type of deafness.
D. Is used to determine the severity of deafness.
E. Is used to determine the side of deafness.
35. Tympanic membrane moves with respiration in:
A. Patulous Eustachian tube.
B. Eustachian tube dysfunction.
C. Otosclerosis.
D. Secretary otitis media.
E. Disconnection of the ossicles.

9
36. Loss of stapedial reflex indicates all of the following except:
A. Severe sensorineural hearing loss.
B. Profound sensorineural hearing loss.
C. Mild sensorineural hearing loss.
D. Facial nerve paralysis.
E. Conductive hearing loss.
37. External auditory canal has……constriction/s:
A. 1.
B. 3.
C. 2.
D. 4.
E. No constriction.
38. Ototoxic drugs involves all of the following except:
A. Gentamicin.
B. Aspirin.
C. Fruesamide.
D. Paracetamol.
E. Cisplatin.
39. A 50 year old male patient presented with otalgia, on examination both external
auditory canal and tympanic membrane are normal, all of the following might
be the sites of origin of his pain except:
A. Neck.
B. Pharynx.
C. Brain.
D. Eye.
E. Nose
40. A nine months old baby has put a plastic bead in his ear. On otoscopy, it is
present in deeper portion of meatus and totally occluding the meatus. The best
way to remove this bead is:
A. Remove it with foreign body hook.
B. Remove it with crocodile forceps.
C. Remove it with suction.
D. Remove it with ear syringing.
E. Remove it under general anesthesia using microscope.
41. A two year child is very slow in developing language. On otoscopy he has dull
ear drum. Investigation of choice is:
A. Pure tone audiogram.
B. X-ray mastoids.
C. Tympanogram.
D. Serum bilirubin.
E. Nystagmogram.

10
42. A 10 year old boy has had ear problems for many years. He has a fever and you
also notice that his mastoid bone is warm and tender to touch. Which of the
following is the best course of action?
A. Refer routinely to ENT.
B. Prescribe topical eardrops.
C. Reassure.
D. Prescribe antibiotics.
E. Refer urgently to ENT.
43. If the patient can hear a whispered voice 30 cm away from the ear, the patient
has:
A. Normal hearing.
B. Sensorineural hearing loss.
C. Conductive hearing loss.
D. Congenital hearing loss.
E. Mixed hearing loss.
44. All of the following tuning fork tests are used to detect organic hearing loss
except:
A. Stenger test.
B. Weber test.
C. ABC test.
D. Rinne test.
E. Modified ABC test.
45. Fistula test is positive in:
A. Acute mastoiditis.
B. Erosion in the basal turn of cochlea.
C. Erosion of the lateral canal.
D. Acute petrositis.
E. Adhesive otitis media.
46. Before bone conduction audiometry masking is done:
A. To prevent cross hearing.
B. To diminish hearing acuity.
C. To increase hearing acuity.
D. To assess speech discrimination.
E. To prevent hair sensation.
47. A blue drum is seen in:
A. Cochlear otosclerosis.
B. Prespyacusis.
C. Vestibular schwannoma.
D. High jugular bulb.
E. Glue ear.
48. The cone of light in the tympanic membrane points:
A. Posteroinferiorly.
B. Superiorly.
C. Anteroinferiorly.
D. Posteriorly.
E. Centrally.

11
49. Ear syringing is contraindicated in patients with:
A. Impacted wax.
B. Otomycosis.
C. Previous history suggesting skull fracture.
D. Presence of foreign body.
E. None of the above.
50. Keratosis obturans is commonly associated with:
A. Bronchiectasis.
B. Appendicitis.
C. Dextrocardia.
D. Renal failure.
E. Diabetes mellitus.

12
Questions for Laryngology
1. Regarding the tonsil, all of the followings are true except:
A. Palatoglossus lies anterior to the tonsil.
B. The tonsil is supplied by the tonsillar artery, a branch of the facial artery.
C. The internal carotid artery lies 2.5cm behind and lateral to the tonsil.
D. Contains 20 crypts.
E. Lymph drains to the upper deep cervical & jugulodigastric lymph nodes.
2. A 66 year old Chinese man presents with a two month history of neck swelling in
the right posterior triangle. On further questioning he tells you that his hearing
has also declined in the right ear. He also has a blocked nose. What is the likely
diagnosis?
A. Lymphoma.
B. Metastatic lymph node from a nasopharyngeal carcinoma.
C. Parotid tumour.
D. Lipoma
E. Metastatic lymph node from a oropharyngeal carcinoma.
3. A 21 year old woman presents with two day history of sore throat and
dysphagia. On examination she is pyrexial with halitosis and cervical
lymphadenopathy. What is the likely causative organism for her condition?
A. Epstein-Barr virus.
B. Agranulocytosis.
C. Corynebacterium diphtheriae.
D. Neisseria gonorrhoea.
E. Group A beta-haemolytic streptococcus (Streptococcus pyogenes).
4. Juvenile angiofibroma:
A. Patients have often had repeated episodes of epistaxis.
B. It is a tumour of young girls with a mean age of 14 at presentation.
C. Bone erosion of the greater wing of the sphenoid does not occur.
D. Secretary otitis media does not occur.
E. External beam radiotherapy is first line treatment.
5. A 55 year old man presents with a three month history of swelling on the right
side of the neck, which has gradually increased in size. He has also been having
earache and throat pain for the last two weeks. He tells you that the pain in his
ear is so severe that he cannot sleep at night. He has no other symptoms. He is
otherwise fit and well. He smokes 10 cigarettes per day and drinks 15 units of
alcohol per week. Examination of the neck reveals a 4 cm x 4 cm firm, mobile,
non-fluctuant, and non-pulsatile swelling on the right side of the neck. The
overlying skin is normal and the mass is not attached to the skin. ENT
examination is normal.
A. Branchial cyst.
B. Malignant node containing squamous cell carcinoma.
C. Infective lymph node.
D. Submandibular gland stone.
E. Non-Hodgkin lymphoma.

13
6. Stridor:
A. It is only a symptom not a sign.
B. Recession of suprasternal, supraclavicular, intercostal and subcostal space
indicate mild form of respiratory difficulty.
C. Cyanosis indicate early stage.
D. Hemangioma can only be congenital.
E. It is never a diagnosis nor a disease.
7. A 27 year old female patient presented with painful and difficulty swallowing,
on indirect laryngoscopy, there is pooling of saliva in right pyriform sinus with
swollen right arytenoid, the most likely diagnosis is:
A. Adenocarcinoma of pyriform sinus.
B. Squamous cell carcinoma of pyriform sinus.
C. Squamous cell carcinoma of posterior pharyngeal wall.
D. Squamous cell carcinoma of larynx.
E. Squamous cell carcinoma of Postcricoid.
8. The first postoperative day, tonsillectomy fossa is:
A. Red colour.
B. Black colour.
C. Yellow colour.
D. White colour.
E. Flesh colour.
9. Stage ІІІ squamous cell carcinoma of oropharynx is:
A. Metastatic disease.
B. Locally advanced disease.
C. Tumour present in lymph nodes.
D. Best treated by radiotherapy.
E. Best treated by chemotherapy.
10. Palliative treatment of advanced head and neck malignant tumours involves all
of the following except:
A. Gastrostomy.
B. Tracheostomy.
C. Pain killers.
D. Nausea and vomiting typically controlled using cyclizine.
E. Antibiotics to treat infections.
11. Radiotherapy is the treatment of choice for:
A. Nasopharyngeal carcinoma.
B. Stage ІІІ squamous cell carcinoma of larynx.
C. Nasopharyngeal adenocarcinoma.
D. Oropharyngeal carcinoma.
E. Hypopharyngeal adenocarcinoma.
12. Commando operation may be used for the treatment of:
A. Nasopharyngeal carcinoma.
B. Stage ІІІ squamous cell carcinoma of larynx.
C. Nasopharyngeal adenocarcinoma.
D. Oropharyngeal carcinoma.
E. Hypopharyngeal carcinoma.

14
13. All of the following may cause tonsillar ulceration except:
A. Diphtheria.
B. Behget’s syndrome.
C. Vincent’s angina.
D. Agranulocytosis.
E. Papilloma.
14. Early post-tonsillectomy complications include all of the following except:
A. Oedema of uvula.
B. Secondary bleeding.
C. Pneumonia.
D. Referred otalgia.
E. Anaesthetic complications.
15. All of the following are true regarding adenoids except:
A. Produces IgM.
B. Attain maximal size between the ages 3 and 8.
C. Produces IgA.
D. Produces IgG.
E. Produces IgD.
16. Laryngomalacia:
A. The larynx is of an exaggerated adult type.
B. The epiglottis is long and wide and folded backward at each lateral edge.
C. The epiglottis is converted into a delta shaped incomplete cylinder.
D. The glossopharyngeal folds are approximated .
E. None of the above.
17. All are causes of congenital strider except:
A. Acute epiglottitis.
B. Laryngeal web.
C. Subglottic stenosis.
D. Laryngomalacia.
E. Vascular anomaly.
18. Trauma to posterior pillar during tonsillectomy causes:
A. Trismus.
B. Bleeding.
C. Infection.
D. Nasal regurgitation.
E. Ankyloglossia.
19. During a hunting trip a bullet affect one of the hunters in his neck by accident,
he developed neck swelling and hoarsed voice, what could be the cause of his
voice change:
A. Unilateral vocal cord palsy.
B. Bilateral vocal cord palsy.
C. Laryngeal oedema.
D. All the above.
E. None of the above.

15
20. Which of the following is part of laryngopharynx:
A. Epiglottis.
B. Cricoids cartilage.
C. Thyroid cartilage.
D. Postcricoid.
E. Arytenoids.
21. Absolute indication of tonsillectomy is:
A. Chronic tonsillitis.
B. Diptheria carrier.
C. Obstructive sleep apnoea .
D. Glassopharyngeal neurectomy.
E. Rheumatic fever.
22. Which of the following is not a complication of acute tonsillitis?
A. Peritonsillar abscess.
B. Bezold’s abscess.
C. Acute rheumatism.
D. Acute nephritis.
E. Acute infection of middle ear cleft.
23. Pyriform fossa lies:
A. Medial to aryepiglottic fold.
B. Lateral to aryepiglottic fold.
C. Inferior to aryepiglottic fold.
D. Superior to aryepiglottic fold.
E. None of the above.
24. A new born baby presented with weak cry and hoarseness, the most probable
diagnosis:
A. Laryngomalacia.
B. Subglotic stenosis.
C. Subglotic heamangioma.
D. Vocal cord palsy.
E. Tracheomalacia.
25. Which of the following is not a feature of pharyngeal diphtheria?
A. Enlarged tender cervical lymph nodes.
B. Pyrexia.
C. True membrane.
D. Toxaemia.
E. Bull's neck.
26. Which of the following is not true regarding acute reteropharyngeal abscess:
A. It is common in children.
B. Produces difficulty in breathing and suckling.
C. It is limited to one side of the midline.
D. Incision and drainage is done from outside through carotid sheath.
E. It forms due to suppuration in reteropharyngeal lymph node

16
27. A 60- year-old man presents to the clinic with a 5 cm solitary lymph node in the
upper right cervical region and an abnormal-looking right tonsil. Assuming
this is a squamous cell carcinoma nodal metastasis, what is the N classification?
A. N1.
B. N2a.
C. N2b.
D. N2c.
E. N3.
28. Subglottic edema can prove fatal in a child because:
A. Mucosa is lax .
B. Mucosa is poorly adherent.
C. Larynx is comparatively small.
D. Subepithelial tissue is lax.
E. All the above.
29. A 70- year-old smokers presents to the ENT clinic with a 3-month history of
dysphagia and weight loss. Flexible nasoendoscopy shows pooling of saliva in the
pyriform fossae, with restricted mobility of the right hemi-larynx. Malignancy is
suspected. You proceed to general anaesthetic endoscopy. At surgery, you find
that he has a 3 cm tumour affecting the right pyriform fossa and extending into
post-cricoid region. According to the TMN staging, what T- stage is this
malignancy?
A. T1.
B. T2.
C. T2B.
D. T3.
E. T4.
30. A 20- year-old man presents to the ENT clinic with sudden onset high fever of
one day duration, on examination: both tonsils are congested with whitish
membrane over the right tonsil with petichae over the palate and palpable both
jugulodiagastric lymph nodes. What is the most possible diagnosis?
A. Acute membranous tonsillitis.
B. Infectious mononucleosis.
C. Diphtheria.
D. Quinsy.
E. Leukaemia.
31. The least complication of tonsillectomy is:
A. Haemorrhage.
B. Pneumonia.
C. Lung abscess.
D. Atelactesis.
E. Subacute bacterial endocarditis.

17
32. All are functions of the larynx except:
A. To protect the lungs.
B. To control air flow.
C. Phonation.
D. Generation of speech.
E. To build positive intrathoracic pressure.
33. Stridor:
A. It is an auditory manifestation of disordered respiratory function due to air
flow changes within the larynx, trachea, or bronchi.
B. It needs investigations in every case.
C. It is due to turbulence of air flow within a partially obstructed respiratory
tract.
D. It can be described in terms of its relationship to the phase of respiratory
cycle.
E. All the above.
34. The most common site of malignant tumours of hypopharynx is:
A. Postcricoid.
B. Cricoids cartilage.
C. Posterior pharyngeal wall.
D. Piriform sinus.
E. Aryepiglottic fold.
35. Which of the following is not true about nasopharyngeal cancer?
A. Metastasizes to cervical lymph nodes.
B. Causes unilateral serous otitis media.
C. Treatment of choice is radiotherapy.
D. EB virus is responsible.
E. Most common nerve involved is vagus.
36. Trismus accompanying quinsy is due to spasm of which muscle?
A. Masseter.
B. Pharyngeal constrictors.
C. Medial pterygoid.
D. Temporalis.
E. Palatopharyngeus.
37. Lymphoid tissue called Waldeyerʼs ring is situated in:
A. Nasopharynx.
B. Upper tow parts of the pharynx.
C. Oropharynx.
D. Base of tongue.
E. Hypopharynx.
38. A 15-year-old has unilateral nasal obstruction, mass in the cheek and recurrent
epistaxis, the diagnosis is:
A. Cancer of nasopharynx.
B. Inverted papilloma nose.
C. Maxillary sinusitis.
D. Bleeding tendency.
E. Angiofibroma.

18
39. A three year old mongol boy get high fever then associated with difficult
swallowing and strider, the child was sitting , cannot sleep and drooling saliva,
the resident pediatrician sent for lateral cervical X-ray then he asked your
opinion about what was looking as a thumb in front of the hypopharynx, your
diagnosis was:
A. Acute pharyngitis.
B. Acute simple laryngitis.
C. Laryngo-tracheo-bronchitis.
D. Acute epiglottitis.
E. Croup.
40. What is not correct in acute epiglottitis?
A. Constant supervision in hospital is mandatory.
B. Dyspnea may be progressing and alarming.
C. It is a special form of acute laryngitis, in which the inflammatory changes affect
mainly the loosely attached mucosa of the epiglottis.
D. Systemic antibiotics is not a must to be started immediately.
E. Age incidence is between 1-6 year old.
41. The mouth:
A. Extends from the incisor teeth to the oropharynx.
B. Contain three salivary glands.
C. Separated from the oropharynx by an anatomical barrier.
D. Consists from two main parts only.
E. The valecula is part of its floor.
42. What is wrong about the pharynx?
A. Extends from base of the skull to the sixth cervical vertebra at upper border of the
cricoid cartilage.
B. It is the upper part of the respiratory tract.
C. It is the upper part of the digestive tract.
D. About 10 cm in length in adult.
E. The pharyngeal cavity opens in front into the nose, mouth, and larynx.
43. The oral vestibule:
A. Located between the teeth and alveoli.
B. Located between the lips and cheek.
C. The space enclosed by the teeth and alveoli .
D. It is site of drainage of salivary gland.
E. It communicates posteriorly with the oropharynx through the oropharyngeal
isthmus.
44. The mylohyoid muscle:
A. It is a depressor of the mandible.
B. It is supplied by maxillary branch of the trigeminal 5th cranial nerve.
C. It is an elevator of the mandible.
D. It is a protractor of the mandible.
E. It is a retractor of the mandible.

19
45. What is true about lymphatic drainage of the vocal cords?
A. It has poor lymphatic drainage.
B. It has rich lymphatic drainage.
C. It has no lymphatic drainage.
D. It has a lymphatic drainage only in the anterior half.
E. None of the above.
46. Thornwaldt’s cyst is seen in:
A. Floor of mouth.
B. Oropharynx.
C. Nasopharynx.
D. Hypopharynx.
E. Larynx.
47. All of the following statements are true about nasopharyngeal tonsil except:
A. They are lined by squamous epithelium.
B. They are present at birth and disappear by puberty.
C. They are situated in the roof and posterior wall of nasopharynx.
D. They do not have capsule.
E. They do not have crypts.
48. Indications of tracheostomy:
A. In all cases of acute laryngitis.
B. In any intubated patient within 3 days.
C. Unilateral choanal atresia.
D. Ludwig's angina .
E. If there is suspicion of laryngomalacia.
49. Causes of aspiration pneumonia include all but:
A. Laryngomalacia.
B. Myasthenia gravis.
C. Polyneuritis.
D. Tetanus .
E. Cervical cord lesion.
50. Care of tracheostomized patient include:
A. Humidification by wet gauze.
B. Changing the tube.
C. Care of the inflatable cuff.
D. All the above.
E. None of the above.

20
Questions for Rhinology
1. Round, smooth, soft, translucent, yellow or pale, glistening structure which
result from prolapsed lining of the ethmoid sinus and blocks the nose to variable
degree depending on their size:
A. Antro choanal polyp.
B. Ordinary nasal polyp.
C. Iinverted papilloma.
D. Furuncolosis.
E. Nasal turbinate.
2. It is an IgE mediated hypersensitivity disease of the mucous membrane of the
nasal air way:
A. Acute rhinitis.
B. Chronic rhinitis.
C. Vasomotor rhinitis.
D. Allergic rhinitis.
E. Furuncolosis.
3. Atopy refers to the tendency to develop an exaggerated IgE antibody response as
reflected by skin prick test in response to one or more of common aeroallergens,
it is extremely common affecting up to :
A. 1/2 of population.
B. 1/3 of population.
C. 1/4 of population.
D. 1/5 of population.
E. 1/6 of population.
4. Blood supply of the nose:
A. Supplied by external carotid artery only.
B. Supplied by internal carotid artery only.
C. There is no connection with the sagittal sinus.
D. The maxillary artery is the main arterial supply to the nasal fossa.
E. The facial artery is a terminal branch of the external carotid artery.
5. Examination of the nose:-
A. Only inferior turbinate can be seen by anterior rhinoscopy.
B. All turbinates can be seen by anterior rhinoscopy.
C. Superior turbinate cannot be seen in children by anterior rhinoscopy.
D. Olfactory epithelium can be seen in children by anterior rhinoscopy.
E. Nasal endoscopy has little to do in routine examination.
6. The lymphatic drainage of the nose:
A. Go directly to the inferior deep cervical lymph nodes.
B. Never pass to the submental lymph nodes.
C. The tip of the nose is devoid of lymphatic drainage.
D. The posterior part of the nose is drained to the retropharyngeal and upper deep
cervical lymph nodes.
E. None of the above.

21
7. Allergic rhinitis:
A. It is an IgG mediated.
B. Rarely associated with bronchial asthma.
C. Productive cough is the main symptom.
D. It is associated with purulent nasal discharge.
E. It is not uncommon disease.
8. Anti-cholinergic (topical ipratrobium bromide) is used in the treatment of:
A. Allergic rhinitis.
B. Eosinophilic vasomotor rhinitis.
C. Non- eosinophilic vasomotor rhinitis.
D. Acute rhinitis.
E. Chronic rhinitis.
9. Allergic rhinitis:
A. It is type 2 allergic hypersensitivity reaction.
B. It involves IgM reaction.
C. The mast cells have a major role in pathogenesis.
D. Neutrophil count is essential for diagnosis.
E. It is the main cause of atopy.
10. The external nose shape is maintained by skeletal framework which is composed
of:
A. Bone only.
B. Cartilage only.
C. Bone in upper 1/3 and cartilage in lower 2/3.
D. Bone in upper 2/3 and cartilage in lower 1/3.
E. Bone in upper 1/2 and cartilage in lower 1/2.
11. The little’s area is supplied by :
A. Superior labial, anterior ethmoidal, greater palatine and spheno-ethmoidal
arteries.
B. Superior labial, posterior ethmoidal, greater palatine and spheno-ethmoidal
arteries.
C. Superior labial, anterior ethmoidal, lesser palatine and spheno-ethmoidal arteries.
D. Inferior labial, anterior ethmoidal, greater palatine and spheno-ethmoidal arteries.
E. None of the above.
12. A ten year old girl presented with pain between the eyes, frontal headache,
discharge from the nose, post nasal drip and high fever; what is the provisional
diagnosis?
A. Acute frontal sinusitis.
B. Acute ethmoidal sinusitis.
C. Acute sphenoidal sinusitis.
D. Sphenoidal tumor.
E. Chronic ethmoidal sinusitis.

22
13. All about the vestibule of the nose are true except:
A. It is the entrance to the nasal cavity.
B. It is lined with skin.
C. It contains sebaceous glands and hairs.
D. It is lined by mucus membrane of respiratory type.
E. Can be seen without nasal speculum.
14. Dorsum of the nose is formed by all except:
A. Septal cartilage.
B. Nasal bones.
C. Upper lateral cartilages.
D. Vomer.
E. Maxillary crest.
15. Mucosa of the nasal cavity is formed by all but:
A. Skin.
B. Columnar ciliated epithelium.
C. Olfactory epithelium.
D. Occasionally squamous epithelium.
E. Respiratory epithelium.
16. Vasomotor rhinitis:
A. It is an infective process.
B. It is an allergic process.
C. It is divided in to two groups: eosinophilic and non-eosinophilic.
D. It is a neoplastic process.
E. It is an infective and allergic process.
17. What is most true about the function of the paranasal sinuses?
A. They protect the eye.
B. Help in olfaction.
C. They reduce the weight of the skull.
D. Do not appear to have a function.
E. They produce resonance of sound.
18. Inflammation of the external nose:
A. Furunolosis is due to streptococcus infection.
B. Furuncolosis cannot be complicated by cavernous sinus thrombosis.
C. Usually painless.
D. Need many investigations for diagnosis.
E. Infections should be treated with antibiotics.
19. Nasal septal hematoma:
A. It is only associated with trauma.
B. It is benign tumour.
C. It is a collection of blood between nasal mucosa and perichondrium.
D. It is mostly unilateral.
E. It is mainly treated by aspiration.

23
20. Regarding deviated nasal septum, which of the following is false?
A. Deviated nasal septum (DNS) is a common physical disorder.
B. It is most frequently caused by impact trauma.
C. Deviated septum is associated with genetic connective tissue disorders such
as Marfan syndrome.
D. Septal haematoma and septal abscess are complications of its correction.
E. Mostly treated surgically.
21. A healthy 15 years male patient presented to the ENT department with right
sided nasal obstruction, foul-smelling and blood-stained discharge. The most
likely diagnosis is:
A. Simple nasal polyp.
B. Rhinolith.
C. Antrochoanal polyp.
D. Foreign body.
E. Angiofibroma.
22. Fractured nose:
A. There are four types.
B. The most common type is type 2.
C. Is the most common facial injuries.
D. Its classification depends on the velocity of trauma only.
E. Diagnosis solely depends on the X-ray finding.
23. Sluder’s neuralgia:
A. Decongestants resolves the pain temporarily.
B. It is synonymous to posterior ethmoid neuralgia.
C. Pain localized on both sides of the face.
D. Due to compression of deviated septum and inferior turbinate.
E. Pain described as dull type.
24. The most common three causative bacterial agents of acute sinusitis are:
A. Streptococcus pneumoniae, Haemophilus influenzae, and Moraxella
catarrhalis.
B. Streptococcus pneumoniae, Staphylococcus aureus, and Moraxella
catarrhalis.
C. Streptococcus pneumoniae, Haemophilus influenzae, and Staphylococcus
aureus.
D. Staphylococcus aureus, Haemophilus influenzae, and Moraxella
catarrhalis.
E. Streptococcus pneumoniae, Haemophilus influenzae, and anaerobic bacteria.
25. Pain due to acute sinusitis:
A. It is not related to the sinus involved.
B. It has a diurnal variation.
C. It is agonizing in open type of sinusitis.
D. Not associated with otalgia.
E. Pain killers alone relieve it.

24
26. All of the following are causes of bilateral nasal obstruction except:
A. Adenoids.
B. Allergic rhinitis.
C. Antrochoanal polyp.
D. Ethmoidal polyp.
E. Atrophic rhinitis.
27. All of the following are complications of sinusitis except:
A. Cavernous sinus septic thrombosis.
B. Meningitis.
C. Toxic shock syndrome.
D. Glomerulonephritis.
E. Pott's puffy tumor.
28. Regarding the management of sinusitis, which of the following is false?
A. Antibiotics are not recommended for most cases.
B. Nasal endoscopy is generally a completely painless procedure which takes
between five to ten minutes to complete.
C. For sinusitis lasting more than 12 weeks a CT scan is recommended.
D. Imaging by either X-ray, CT or MRI is generally not recommended in acute
sinusitis unless complications develop.
E. Acute sinusitis mostly treated by surgery.
29. Nasal foreign body:
A. Spherical foreign body removed by Telly nasal dressing forceps.
B. Epistaxis is the most common symptom.
C. Chargeable battery should be urgently remove.
D. Only occur in children.
E. Usually bilateral.
30. Local precipitating factor for maxillary sinusitis:
A. Poor diet.
B. Fatigue.
C. Nasal obstruction.
D. Chilling.
E. Irritating atmospheric conditions.
31. Definite diagnosis of acute maxillary sinusitis:
A. Facial pain.
B. Antral lavage.
C. Pain in the cheek.
D. Tenderness over the cheek.
E. X-ray- water’s view.
32. The most common cause of epistaxis is:
A. Hypertension.
B. Idiopathic.
C. Fracture nasal bone.
D. Blood diseases.
E. Angiofibroma.

25
33. All are true about anosmia except:
A. It can be tested by simple objective method.
B. It must be bilateral before it is noticeable.
C. It is often described as loss of taste.
D. It may be due to brain tumor.
E. Trauma is not a cause of it.
34. A complication of common cold:
A. Otitis externa.
B. Quinsy.
C. Facial palsy.
D. Maxillary sinusitis.
E. Inverted papilloma.
35. Symptoms of acute staphylococcus infection of nasal vibrissae include all but:
A. Pain.
B. Fever.
C. Swelling.
D. Nasal obstruction.
E. Headache.
36. Mast cell stabilizers:
A. Can be used in treatment of all kinds of allergic disease.
B. Usually used as the main treatment line in allergic rhinitis patients.
C. Mostly used as prophylactic in seasonal rhinitis.
D. Used as one of the cornerstones in the treatment of allergic rhinitis.
E. All the above.
37. The antrochoanal polyp:
A. Is uncommon.
B. It’s mostly unilateral.
C. Extends immediately to the posterior choana.
D. All the above.
E. None of the above.
38. The main problem in using immunotherapy (hypo-sensitization) in allergic
rhinitis patient is:
A. The cost.
B. Risk of hypersensitivity reaction.
C. Difficulty with identification of the antigen.
D. The technique.
E. All the above.
39. All structures open in the middle meatus except:
A. Sphenoid sinus.
B. The maxillary sinus.
C. The frontal sinus.
D. The anterior ethmoid air cells.
E. The middle ethmoid air cells.

26
40. Stages of common cold include all but:
A. Ischemic stage .
B. Hyperemic stage.
C. Stage of secondary infection.
D. Stage of resolution.
E. Febrile stage.
41. Treatment of common cold include all but:
A. Steam inhalation.
B. Warm fluid.
C. Antibiotics.
D. Bed rest.
E. Analgesia.
42. Which of the following is not a symptom of acute maxillary sinusitis?
A. Pain in the cheek.
B. Tenderness over the cheek.
C. Edema over the cheek.
D. Discharge from the middle meatus.
E. Dullness on trans-illumination.
43. Best radiological test for maxillary sinus is:
A. Occipito-frontal view.
B. Occipito-mental view.
C. Lateral view.
D. CT scan.
E. Submento-vertical view.
44. Best section in CT scan for maxillary sinus is:
A. Axial.
B. Coronal.
C. Three dimensional.
D. Sagittal.
E. Transverse.
45. In seasonal rhinitis, the early symptom will be:
A. Paroxysms of sneezing occur at frequent intervals throughout the day.
B. Watery rhinorrhea.
C. Loss of smell and loss of taste.
D. Post nasal drip is not common.
E. Facial pain.
46. Best treatment line in allergic rhinitis is:
A. Local antihistamine.
B. Local corticosteroid.
C. Systemic corticosteroid.
D. Allergen avoidance.
E. Local vasoconstrictor.

27
47. In perennial rhinitis the commonest cause is:
A. Grass pollen.
B. Tree pollen.
C. Fungi spores.
D. House dust mites.
E. Flour dust.
48. The best investigation in antrochoanal Polyp:
A. Skin brick test.
B. Waters view X-ray.
C. Nasal endoscopy.
D. Coronal CT scan.
E. All the above.
49. The antrochoanal polyp:
A. The patient, commonly a young adult, complains of bilateral nasal obstruction.
B. The patient, commonly a young adult, complains of unilateral nasal obstruction
which is worse on inspiration.
C. The patient, commonly an old female, complains of unilateral nasal obstruction
which is worse on expiration.
D. All the above
E. None of the above.
50. Not a cause of acute maxillary sinusitis:
A. Acute infective rhinitis.
B. Common cold.
C. Swimming.
D. Fracture of maxillary sinus.
E. Barotrauma.

28
Part 2 – The answers
Answers of Otology questions
1. A
Cholesteatoma is a destructive and expanding growth consisting of keratinizing
squamous epithelium in the middle ear and/or mastoid process. Although these
are not strictly speaking tumours or cancers they can still cause significant
problems because of their erosive and expansile properties resulting in the
destruction of the ossicles as well as their possible spread through the base of the
skull into the brain. They are also often infected and result in chronically draining
ears. It must be treated surgically unless the patient refuse the surgery or elderly
with significant medical problems.
2. D
Development of the ear, the complex organ of hearing and balance, involves all 3
embryonic germ layers
1. ECTODERM is the origin of the internal and external ears
2. ENDODERM takes part in the formation of the middle ear
3. MESODERM plays a role in the formation of all 3 parts of the ear.
Regarding the tympanic membrane, the outer squamous layer is derived
from ectoderm and the inner layer from endoderm while the middle
connective tissue layer is derived from mesoderm.
3. B
The footplate of the stapes helps with hearing. The stapes bone notifies the oval
window when movement occurs. The stapes footplate proceeds into the oval
window, sending the round window membrane out, allowing for the fluid in the
cochlea to move, which leads to the cochlear inner hair cells’ movement, and
finally, hearing.
4. A
The auricle (pinna) attains 90-95% of adult size by5-6 years. Due to this fact
otoplasty only performed around the age 6 years onward.
5. B
A Mastoidectomy is a procedure performed to remove the mastoid air cells. This
can be done as part of treatment for mastoiditis, chronic suppurative otitis media
or Cholesteatoma. In addition, it is sometimes performed as part of other
procedures (cochlear implant) or for access to the middle ear. There are
classically 5 different types of Mastoidectomy:
1. Radical Mastoidectomy - Removal of posterior and superior canal wall,
meatoplasty and exteriorisation of middle ear.
2. Canal Wall Down Mastoidectomy - Removal of posterior and superior
canal wall, meatoplasty. Tympanic membrane left in place.
3. Canal Wall Up Mastoidectomy - Posterior and superior canal wall are kept
intact. A facial recess approach is taken.
4. Cortical Mastoidectomy (Also known as schwartze procedure) - Removal
of Mastoid air cells is undertaken without affecting the middle ear. This is
typically done for mastoiditis
5. Modified Radical Mastoidectomy - This is confusing because it is
typically described as a radical mastoidectomy while maintaining the

29
posterior and superior canal wall which reminds the reader of the Canal
Wall Up Mastoidectomy. However, the difference is historical. Modified
radical mastoidectomy typically refers to Bondy’s procedure which
involves treating disease affecting only the epitympanum. Diseased areas
as well as portions of the adjacent superior and posterior canal are simply
exteriorised without affecting the uninvolved middle ear.
6. C
The best treatment for glue ear is consume time by watchful technique. In most
children, episodes of glue ear get better without active treatment. For this reason,
Otolaryngologist may simply recommend regular appointments for up to three
months so they can check the ears. If he finds a fluid in the space behind the ear
drum for more than three months, the beast treatment option is myringotomy with
ventilation tube (Grommets) insertion.
7. E
The auricular branch of the vagus nerve is often termed the Alderman's nerve or
Arnold’s nerve. The latter name is an eponym for Friedrich Arnold. It supplies
sensory innervation to the skin of the ear canal. In a small portion of individuals,
the auricular nerve is the afferent limb of the Ear-Cough or Arnold Reflex.
Physical stimulation of the external acoustic meatus innervated by the auricular
nerve elicits a cough, much like the other cough reflexes associated with the vagus
nerve. Rarely, on introduction of speculum in the external ear, patients have
experienced syncope due to the stimulation of the auricular branch of the vagus
nerve.
8. A
Prespyacusis is hearing loss occurring in old age individuals due progressive
degeneration in the auditory system with ageing, which leads to hearing impairment
in the affected individual as with all sensory systems in the human body.
Presbyacusis is defined as the lessening of the acuteness of hearing that characterizes
old age. Tympanosclerosis, otosclerosis, longitudinal fracture of the temporal bone
and cholesteatoma are the causes of conductive deafness.

9. D
The brainstem auditory evoked response (BAER) has proved useful in determining
the hearing threshold in neonates and even very young uncooperative patients. but
although still a research tool for the investigation of cochlear function, the use of evoked
otoacoustic emissions (OAEs) has now found a place in clinical practice in the screening
of neonates and high-risk infants for hearing loss. Evoked OAEs are quick, easy to test
and do not require an anaesthetic, in contrast to electrical evoked response audiometry.
The sensitivity and specificity of the test is sufficiently good that there are widespread
recommendations that all newborn infants are screened for hearing loss by OAE prior to
discharge from hospital.
10. D
A siegel’s pneumatic speculum has an eye piece which has a magnification of 4-5 times.
It is a convex lens. The eye piece is connected to an aural speculum. A bulb with a rubber
tube is provided to insufflate air via the aural speculum .The advantages of this aural
speculum is that it provides a magnified view of the ear drum, the pressure of the external

30
canal can be varied by pressing the bulb thereby the mobility of ear drum can be tested.
Since it provides adequate suction effect, it can be used to suck out middle ear secretions
in patients with CSOM. Ear drops or powder can be applied into the middle ear by using
this speculum. Ear is first filled with ear drops or powder and a snugly fitting siegel's
speculum is applied to the external canal. Pressure in the external canal is varied by
pressing and releasing the rubber bulb, this displaces the ear drops into the middle ear
cavity. A siegel's pneumatic speculum has an ability in detecting a fistula in inner ear
(fistula is due to an erosion in bony labyrinth with an intact endosteum) ) by applying
pressure in the middle ear there will be irritation of the labyrinth leading to nausea
,vomiting, vertigo and nystagmus and the result is said to be positive fistula test.
Sometimes a fistula present but the test is negative ,this occur in dead labyrinth and the
result is said to be false negative.
11. A
The syndrome, first described by Gradenigo in 1907, consists of the clinical triad of acute
otitis media, unilateral pain in regions innervated by the first and second branch of the
trigeminal nerve, and ipsilateral abducens nerve paralysis. These cranial nerve
dysfunctions are caused by osteitis of the petrous apex (petrous apicitis) and are very rare
complications of otitis media, especially since the widespread use of antibiotics.
12. A
The Weber test is a quick screening test for hearing. It can detect unilateral (one-
sided) conductive hearing loss (middle ear hearing loss) and unilateral sensorineural
hearing loss (inner ear hearing loss). The test is named after Ernst Heinrich Weber
(1795–1878).
A patient with a unilateral conductive hearing loss would hear the tuning fork loudest in
the affected ear. This finding is because the conduction problem of the middle ear (incus,
malleus, stapes, and Eustachian tube) masks the ambient noise of the room, while the
well-functioning inner ear (cochlea with its basilar membrane) picks the sound up via the
bones of the skull, causing it to be perceived as a louder sound in the affected ear.
Another theory, however, is based on the occlusion effect described by Tonndorf et al. in
1966. Lower frequency sounds (as made by the 256 Hz fork) that are transferred through
the bone to the ear canal escape from the canal. If an occlusion is present, the sound
cannot escape and appears louder on the ear with the conductive hearing loss.
13. A
Malignant (necrotizing) otitis externa (MOE) was first described as a case of
progressive Pseudomonas osteomyelitis in the temporal bone of elderly patient who had
diabetes nearly a half century ago.
Any condition causing immunosuppression, including HIV/AIDS, chemotherapy-
induced aplasia, refractory anemia, chronic leukemia, lymphoma, splenectomy,
neoplasia, and renal transplantation, may predispose a patient to MOE.
MOE should be suspected in all immunocompramized patients who have otitis externa
that does not improve with appropriate therapy. MOE have typical granulation tissue
along the floor of the external auditory canal (EAC).
Although its name is malignant but it is not neoplastic condition and the mitotic figures
remain normal ,the name (malignant) describes the aggressiveness and prognosis of the
condition.

31
14. D
An otoacoustic emission (OAE) is a sound which is generated from the outer hair
cells within the inner ear. Having been predicted by Thomas Gold in 1948, its existence
was first demonstrated experimentally by David Kemp in 1978 and otoacoustic emissions
have since been shown to arise through a number of different cellular and mechanical
causes within the inner ear. Studies have shown that OAEs disappear after the inner ear
has been damaged, so OAEs are often used in the laboratory and the clinic as a measure
of inner ear health. Using modern computing technology and signal averaging
techniques, outer hair cell vibrations can be detected in the external auditory meatus as
otoacoustic emissions (OAEs).It represents an objective measure of cochlear function.
Acoustically evoked OAEs are almost never found in ears with a hearing level worse than
40 dB.
15. A
Human ears can hear sound frequencies from 20-20000 Hz, But most of our speech
lies in range of 500-2000Hz.
16. C
Bone conduction decreases in sensorineural hearing loss and not in conductive
hearing loss, all the above conditions cause condutive hearing loss except cochlear
damage which causes sensorineural hearing loss.
In sensorineural hearing loss both air and bone conduction curves are decreased and the
difference between them <15 dB.
17. B
A perforated eardrum or punctured eardrum is a rupture or perforation (hole) of the
eardrum which can occur as a result of otitis media (ear infection), trauma (e.g. by trying
to clean the ear with sharp instruments), explosion, loud noise or surgery (accidental
creation of a rupture). Flying with a severe cold can also cause perforation due to changes
in air pressure and blocked Eustachian tubes resulting from the cold. This is especially
true on landing.
Perforation of the eardrum leads to conductive hearing loss, which is usually
temporary. Other symptoms may include tinnitus, earache or a discharge of mucus.
Traumatic eardrum perforation usually has an irregular shape while infective
perforation has regular shape.
18. D
The caloric reflex test (sometimes termed 'vestibular caloric stimulation') is a test of
the vestibulo-ocular reflex that involves irrigating cold or warm water or air into the
external auditory canal.
Ice cold or warm water or air is irrigated into the external auditory canal, usually using
a syringe. The temperature difference between the body and the injected water creates a
convective current in the endolymph of the nearby horizontal semicircular canal. Hot and
cold water produce currents in opposite directions and therefore a horizontal nystagmus
in opposite directions. In patients with an intact brainstem: If the water is warm (44°C or
above) endolymph in the ipsilateral horizontal canal rises, causing an increased rate of
firing in the vestibular afferent nerve. This situation mimics a head turn to the ipsilateral
side. Both eyes will turn toward the contralateral ear, with horizontal nystagmus to the
ipsilateral ear.

32
If the water is cold, relative to body temperature (30°C or below), the endolymph falls
within the semicircular canal, decreasing the rate of vestibular afferent firing. The eyes
then turn toward the ipsilateral ear, with horizontal nystagmus (quick horizontal eye
movements) to the contralateral ear.
19. D
When a fresh eardrum perforation is immediately confirmed, it is important to
instruct the patient to keep contaminated or soapy water out of the ear canal and to avoid
blowing the nose and elevation changes, all of which can serve to either create an
infection or maintain an open perforation.
20. D
Earwax, also known by the medical term cerumen, is a yellowish waxy substance
secreted in the ear canal of humans and other mammals. It protects the skin of the human
ear canal, assists in cleaning and lubrication, and also provides some protection from
bacteria, fungi, insects and water. Excess or impacted cerumen can press against the
eardrum and/or occlude (block) the external auditory canal or hearing aids, potentially
hindering hearing.
Cerumen is produced in the outer third of the cartilaginous portion of the ear canal. It is
a mixture of viscous secretions from sebaceous glands and less-viscous ones from
modified apocrine sweat glands. The primary components of earwax are shed layers of
skin, with 60% of the earwax consisting of keratin, 12–20% saturated and unsaturated
long-chain fatty acids, alcohols, squalene and 6–9% cholesterol.
21. D
Normal person: Type A tympanogram
Tympanosclerosis: Type As tympanogram
Otosclerosis: Type As tympanogram
Secretary otitis media: Type B tympanogram
Disconnection of the ossicles: Type Ad tympanogram
22. A
Types of intracranial complications of chronic suppuration otitis media:
1. Extradural abscess
2. Subdural abscess
3. Meningitis
4. Brain abscess
5. Lateral sinus thrombophlebitis.
6. Otitic hydrocephalus
Types of extracranial complications of chronic suppuration otitis media:
1. Mastoiditis
2. Petrositis
3. Labyrinthitis
4. Facial paralysis
5. Mastoid abscesses.
6. Squamous cell carcinoma of the ear.
23. C
Otalgia or an earache is pain in the ear. Primary otalgia is ear pain that originates
inside the ear. Referred otalgia is ear pain that originates from outside the ear.
Otalgia is not always associated with ear disease. It may be caused by several other

33
conditions, such as impacted teeth, sinus disease, inflamed tonsils, infections in the
nose and pharynx, throat cancer, and occasionally as a sensory aura that precedes a
migraine.
Allergic rhinitis is usually not associated with pain unless there is an infection of the
paranasal sinuses.
24. A
Acute mastoiditis symptoms may include:
1. Fever, irritability, and lethargy.
2. Swelling of the ear lobe.
3. Redness and tenderness behind the ear with obliteration of postauricular
sulcus.
4. Drainage from the ear.
5. Bulging and drooping of the ear.
One of the signs is obliteration of the postauricular sulcus which
differentiate it from otits externa in which this sulcus become widened.
25. A
The ratio of the functioning area of the tympanic membrane to the area of the
footplate is 14:1.
26. B
The primary goal of surgery for CSOM is make the ear safe (less complications
especially the intracranial complications).
27. A
 Otitis media with effusion is the most common cause of bilateral conductive
deafness in children.
 Otosclerosis is a cause of bilateral conductive deafness but in young adult and is
rare in children.
 Acute otitis media is common in young children, might cause a unilateral or
bilateral and not causing persistent deafness unless it progress to chronic state.
 Congenital cholesteatoma is rare cause of conductive deafness and mostly
unilateral.
 Chronic suppurative otitis media: might cause unilateral or bilateral conductive
deafness but allover is less in occurrence than Otitis media with effusion.
28. C
Cochlear implant is the final solution for such child and should be done before the
brain loses the neural plasticity at age of 6-8 years.
29. A
All are causes of peripheral episodic vertigo but benign paroxysmal positional
vertigo is the most common cause.
30. D
Meticulous and regular aural toilet paying particular attention to the anteroinferior
meatal recess plus antifungal ear drops is the best option for otomycosis.
31. C
Indications for Adenoidectomy
1. Nasal obstruction.
2. Otitis media with effusion (glue car).
3. Recurrent acute otitis media.

34
4. Chronic rhinosinusitis.
5. Sleep apnea syndrome.
32. C
Complications of mumps are potentially serious, but rare. Most mumps
complications involve inflammation and swelling in some part of the body, such as:
1. Testicles. Orchitis.
2. Pancreas. Pancreatitis.
3. Ovaries and breasts. oophoritis or mastitis. Fertility is rarely affected.
4. Brain. Encephalitis
5. Membranes and fluid around the brain and spinal cord. Meningitis.
Other complications
6. Hearing loss. In rare cases, mumps can cause hearing loss, usually permanent, in
one or both ears.
7. Miscarriage
33. D
34. A
Pure tone audiometry (PTA) is the key hearing test used to identify hearing threshold
levels of an individual, enabling determination of the degree, type and configuration of a
hearing loss. Thus, providing the basis for diagnosis and management. PTA is a
subjective, behavioural measurement of hearing threshold, as it relies on patient response
to pure tone stimuli. Therefore, PTA is used on adults and children old enough to
cooperate with the test procedure. As with most clinical tests, calibration of the test
environment, the equipment and the stimuli to ISO standards is needed before testing
proceeds. PTA only measures thresholds, rather than other aspects of hearing such as
sound localization. However, there are benefits of using PTA over other forms of hearing
test, such as click auditory brainstem response. PTA provides ear specific thresholds, and
uses frequency specific pure tones to give place specific responses, so that the
configuration of a hearing loss can be identified. As PTA uses both air and bone
conduction audiometry, the type of loss can also be identified via the air-bone gap.
35. A
Patulous Eustachian tube, also known as patent Eustachian tube, is the name of a
physical disorder where the Eustachian tube, which is normally closed, instead stays
intermittently open. When this occurs, the patient experiences autophony, the hearing of
self-generated sounds.
Upon examination of a suspected case of patulous Eustachian tube, a doctor can directly
view the tympanic membrane with a light and observe that it vibrates with every breath
taken by the patient.
36. C
The acoustic reflex (also known as the stapedius reflex, attenuation reflex, or auditory
reflex) is an involuntary muscle contraction that occurs in the middle ear of mammals in
response to high-intensity sound stimuli.
When presented with a high-intensity sound stimulus, the stapedius and tensor tympani
muscles of the ossicles contract. The stapedius stiffens the ossicular chain by pulling the
stapes (stirrup) of the middle ear away from the oval window of the cochlea and the

35
tensor tympani muscle stiffens the ossicular chain by loading the eardrum when it pulls
the malleus (hammer) in toward the middle ear. The reflex decreases the transmission of
vibrational energy to the cochlea, where it is converted into electrical impulses to be
processed by the brain. The acoustic reflex normally occurs only at relatively high
intensities; activation for quieter sounds can indicate ear dysfunction. The pathway
involved in the acoustic reflex is complex and can involve the ossicular chain (malleus,
incus and stapes), the cochlea (organ of hearing), the auditory nerve, brain stem, facial
nerve and other components. Consequently, the absence of an acoustic reflex, by itself,
may not be conclusive in identifying the source of the problem.
37. C
The external auditory canal, an inch in length, extends from the concha to the eardrum
& consist of two parts. The outer one third is made of cartilage & called cartilaginous
part. The inner two thirds is made of bone & called the bony part. During its course, it
describes a sigmoid course. Its outer third goes medially, forwards & slightly upwards. Its
middle third courses medially, backwards, & slightly upwards. Its inner third traverses
medially, forwards & downwards. The cross-section of the canal is oval with its long axis
directed downwards & backwards & measures 9-10mm. This is, however, reduced
considerably at the isthmus, a point which is half a centimeter from the ear drum. And
also at the junction of the cartilaginous & bony parts.
38. D
Ototoxicity is the quality of being toxic to the ear (oto-), specifically the cochlea or
auditory nerve and sometimes the vestibular system; it is commonly medication-induced.
Ototoxic drugs include antibiotics such as gentamicin, loop diuretics such as furosemide
and platinum-based chemotherapy agents such as cisplatin. A number of nonsteroidal
anti-inflammatory drugs (NSAIDS) have also been shown to be ototoxic. This can result
in sensorineural hearing loss, dysequilibrium, or both. Either may be reversible and
temporary, or irreversible and permanent.
39. C
The brain has no pain receptors.
40. E
You should remove it under general anesthesia using microscope to avoid acute
complications of ear foreign body removal include canal abrasions, bleeding,
infection, and perforation of the tympanic membrane.
41. C
Pure tone audiogram cannot performed in such age.
This child has secretory otitis media and Type B tympanogram may reveal fluid in
the middle ear.
X-ray mastoids, serum bilirubin and nystagmogram are useless investigations for
such child.
42. E
Mastoid abscess remains a recognized complication of otitis media despite the advent
of antibiotics and need to refer urgently to ENT department.
43. A
The easiest & best method of performing monaural free-field voice testing is by
using a whispered voice, conversational voice & then the loud voice at 60cm & then

36
15cm. The non-test ear is masked by a tragal rub unless a loud voice is required(use the
Barany noise box).
If the patient can hear a whispered voice 60cm away from the ear, the pure tone
thresholds are likely to be less than 30dB(normal hearing). Patients who can hear a
whisper at 15cm or a conversational voice at 60cm or 15cm are likely to have thresholds
in the range of 30-70dB hearing loss(mild to moderate impairment). Those patients who
can only hear a loud voice are likely to have thresholds greater than 70dB hearing loss
(severe/profound impairment).
44. A
A Stenger test for non-organic hearing loss. If two identical tuning forks are used & one
is presented to each ear of normal subject, he can only perceive the nearer of the two
forks. If the examiner holds the two forks behind the blind folded patient & places one
fork about 25cm from the good ear, the patient will say he hears the sound. Next the other
fork is brought to about 8 cm from the ear under test. A patieni with true deafness will
still hear the fork placed 25cm from his good ear, but the patient with non-organic
deafness will deny that he hears any sound at all.
45. C
46. A
There are many different mechanisms of masking, one being suppression. This is when
there is a reduction of a response to a signal due to the presence of another. This happens
because the original neural activity caused by the first signal is reduced by the neural
activity of the other sound.
47. D
High jugular bulb is defined as that showing an abnormally high position in the
tympanic cavity. A high jugular bulb can be seen through the eardrum in some instances.
When the eardrum is not translucent, however, it is impossible to detect a high jugular
bulb by otoscopic examination alone.
48. C
A cone of light can be seen radiating from the tip of the malleus to the periphery in the
antero-inferior quadrant.
49. C
Syringing is contraindicated following recent injury or in patients who have had a
perforated ear drum. It is also contraindicated if there is a history of recent otitis externa
or otitis media, previous middle ear/mastoid surgery, and if it is the only hearing ear. In
these cases the patient should not be syringed and should be referred for dewaxing under
direct vision.
50. A

37
Answers of Laryngology questions
1. D
The tonsils are paired secondary lymphatic organs situated on the side of the
oropharynx between the palatoglossal (anterior tonsillar pillar) & palatopharyngeal folds
(posterior tonsillar pillar). They are part of Waldeyer's ring, a ring of lymphoid tissue
consisting of the adenoids, the tubal tonsils, the palatine tonsils & the lingual tonsils,
which are embedded in the posterior third of the tongue. The tonsil is enclosed by a
fibrous capsule, outside of which is a layer of areolar tissue. This separates the capsule
from the pharyngobasilar fascia covering the superior constrictor muscle that forms the
tonsil bed. Running through the mucosa of each tonsil are 12-15 pits, called crypts . The
main blood supply of the tonsil is from the tonsillar branch of the facial artery.
2. B
Nasopharynx cancer or nasopharyngeal carcinoma (NPC) is the most common
cancer originating in the nasopharynx, the uppermost region of the pharynx ("throat"),
behind the nose where the nasal passages and auditory tubes join the remainder of the
upper respiratory tract. NPC occurs in children and adults. NPC differs significantly from
other cancers of the head and neck in its occurrence, causes, clinical behavior, and
treatment. It is vastly more common in certain regions of East Asia and Africa than
elsewhere, with viral, dietary and genetic factors implicated in its causation. It is most
common in males. It is a squamous cell carcinoma or an undifferentiated type. Squamous
cells are a flat type of cell found in the skin and the membranes that line some body
cavities. Differentiation means how different the cancer cells are from normal cells.
Undifferentiated is a word used to describe cells that do not have their mature features or
functions.
Cervical lymphadenopathy (disease or swelling of the lymph nodes in the neck) is the
initial presentation in many patients, and the diagnosis of NPC is often made by lymph
node biopsy. Symptoms related to the primary tumor include trismus, pain, otitis media,
nasal regurgitation due to paresis (loss of or impaired movement) of the soft palate,
hearing loss and cranial nerve palsies (paralysis). Larger growths may produce nasal
obstruction or bleeding and a "nasal twang". Metastatic spread may result in bone pain or
organ dysfunction. Rarely, a paraneoplastic syndrome of osteoarthropathy (diseases of
joints and bones) may occur with widespread disease.
3. E
Streptococcal pharyngitis, streptococcal tonsillitis, or streptococcal sore throat (known
colloquially as strep throat) is a type of pharyngitis caused by a group A streptococcal
infection. It affects the pharynx including the tonsils and possibly the larynx. Common
symptoms include fever, sore throat, and enlarged lymph nodes. It is the cause of 37% of
sore throats among children and 5-15% in adults.
Strep throat is a contagious infection, spread through close contact with an infected
individual. A definitive diagnosis is made based on the results of a throat culture.
However, this is not always needed as treatment may be decided based on symptoms. In
highly likely or confirmed cases, antibiotics are useful to both prevent complications and
speed recovery. Potential complications include rheumatic fever.

38
4. A
 It most commonly affects adolescent males.
 Nasopharyngeal angiofibroma (also called juvenile nasopharyngeal angiofibroma)
is a histologically benign but locally aggressive vascular tumor that grows in the
back of the nasal cavity.
 Conductive hearing loss from eustachian-tube obstruction
 Patients with tumors that have extended into the cranial cavity or whose tumors
can't be safely reached by surgery may receive radiation therapy. Also Radiation
therapy helps in case where recurrence is the main problem.
5. B
 Metastatic squamous neck cancer with occult primary is a disease in which
squamous cell cancer spreads to lymph nodes in the neck and it is not known
where the cancer first formed in the body.
 Possible signs of metastatic squamous neck cancer with occult primary include
a lump or pain in the neck or throat.
 Tests that examine the tissues of the neck, respiratory tract, and upper part of
the digestive tract are used to detect (find) and diagnose metastatic squamous
neck cancer and the primary tumor.

39
6. E
Stridor is an auditory manifestation of disordered respiratory function due to air flow
changes within the larynx , trachea ,or bronchi. It’s a symptom or sign of many
conditions.
7. E
Causes for pooling of saliva in the pyriform fossa:
Pooling of saliva in the pyriform fossa is not only caused by growth affecting this area
causing obstruction to saliva being swallowed, but also due to intense cricopharyngeal
muscle spasm.
1. Malignant growth involving the deep portion of the pyriform fossa
2. Foreign body being lodged in the pyriform fossa.
3. Growth involving the crico pharynx (Postcricoid) or upper oesophagus can also cause
pooling of saliva.
8. D
The throat will have a thick white coating where the tonsils used to be. This is the
normal healing process and does not indicate infection.
9. C
Stages of squamous cell carcinoma of oropharynx
Stage 0 carcinoma in situ
Abnormal cells are found in the lining of the oropharynx, These may become cancer and
spread into nearby normal tissue.
Stage 1
Cancer has formed and is 20 millimetres or smaller and has not spread outside the
oropharynx.
Stage 2
Cancer has formed and is larger than 20 millimetres but not larger than 40 millimetres.
Also it has not yet spread outside the oropharynx.
Stage 3
 Cancer is larger than 40 millimetres and has not spread outside the oropharynx
 Any size and has spread to only one lymph node on the same side of the neck as
the cancer. The lymph node with cancer is 30 millimetres or smaller.
Stage 4A
 Cancer has spread to tissues near the oropharynx, including the voice box, roof of
the mouth, lower jaw, muscle of the tongue or central muscles of the jaw and may
have spread to one or more nearby lymph nodes; none larger than 60 millimetres.
 Cancer is any size and has spread to one lymph node that is larger than 30
millimetres but not larger than 60 millimetres on the same side of the neck as the
cancer or to more than one lymph node, none larger than 60 millimetres, on one
of both sides of the neck.
Stage 4B
 Cancer surrounds the main artery in the neck or has spread to bones in the jaw or
skull, to muscle in the side of the jaw or to the upper part of the throat behind the
nose and may have spread to nearby lymph nodes
 Cancer has spread to a lymph node that is larger than 60 millimetres and may
have spread to tissues around the oropharynx.

40
Stage 4C
Cancer has spread to other parts of the body; the tumor may be any size and may have
spread to lymph nodes.
10. E
The following are some of the most common potential problems which can arise in
the last days and hours of a patient's life:
1. Pain -Suffering from uncontrolled pain is a significant fear of those at end
of life. Typically controlled using morphine or diamorphine; or other
opioids.
2. Agitation Delirium, terminal anguish, restlessness (e.g. thrashing,
plucking, or twitching). Typically controlled using midazolam, or other
benzodiazepines. Symptoms may also sometimes be alleviated by
rehydration, which may reduce the effects of some toxic drug metabolites.
3. Respiratory Tract Secretions Saliva and other fluids can accumulate in the
oropharynx and upper airways when patients become too weak to clear
their throats, leading to a characteristic gurgling or rattle-like sound
("death rattle"). While apparently not painful for the patient, the
association of the symptom with impending death can create fear and
uncertainty for those at the bedside. The secretions may be controlled
using drugs such as scopolamine (hyoscine), glycopyrronium, or atropine.
Rattle may not be controllable if caused by deeper fluid accumulation in
the bronchi or the lungs, such as occurs with pneumonia or some tumours.
4. Nausea and vomitingTypically controlled using cyclizine; or other anti-
emetics.
5. Dyspnea (breathlessness) Typically controlled using morphine or
diamorphine.
11. A
Radiotherapy: The method of choice for nasopharyngeal carcinoma, because surgical
removal of the primary growth is rarely possible; metastases are often present when the
patient is first seen; and the tumours are usually anaplastic and highly radiosensitive.
12. D
The commando procedure is done for cancers affecting the oropharyngeal region.
This term is coined ‘commando procedure’ as it’s a surgical attack on the diseased
tissue. The head and neck surgery involves resection of the mandible with the floor of the
mouth and the dissection of the entire lymphatic system of the neck along with the
sternoclidomastoid muscle and internal jugular vein.
13. E
Differential diagnosis of ulceration of the tonsil
A working diagnosis can usually be determined from the history & clinical examination.
Investigations include a full blood count, chest radiograph, serological tests & biopsy.
Possible causes include:
1. Infection.
• Acute streptococcal tonsillitis.
• Diphtheria.
• Infectious mononucleosis.
• Vincent's angina.

41
2. Neoplasm.
• Squamous cell carcinoma.
• Lymphoma.
• Salivary gland tumours (adenoid cystic carcinoma).
3. Blood diseases.
• Agranulocytosis.
• Leukaemia.
4. Other causes.
• Aphthous ulceration.
• Behget's syndrome.
• Acquired immunodeficiency syndrome (AIDS).
14. E
Early post-tonsillectomy complications include
• Secondary haemorrhage.
• Haematoma and oedema of the uvula.
• Infection (may lead to secondary haemorrhage).
• Earache (referred pain or acute otitis media).
• Pulmonary complications (pneumonia and lung abscess are rare).
• Subacute bacterial endocarditis (if the patient has a cardiac defect).
Anaesthetic complications might be included in a peroperative or immediate post-
tonsillectomy complications.
15. E
The adenoids are a mass of lymphoid tissue found at the junction of the roof &
posterior wall of the nasopharynx. They are a normal structure with a function in the
production of antibodies (lgA, IgG & IgM). The size of the adenoids varies, but in
general they attain their maximal size between the ages of 3 & 8 years & then regress.
16. E
The larynx is of an exaggerated infantile type. The epiglottis is long and narrow and
folded backward at each lateral edge. This convert the epiglottis into an omega shaped
incomplete cylinder. The aryepiglottic folds are also approximated. The laryngeal inlet is
therefore reduced to a cruciate slit the edges of which sucked together by each
inspiration.
17. A
Acute simple laryngitis is an acute superficial inflammation of the laryngeal mucus
membrane. Which is an acquired and not a congenital condition.
18. D
One of the functions of the posterior pillars with the soft palate is to close the
velopharyngeal isthmus during swallowing and trauma to it can lead to nasal
regurgitation.
19. D
Hoarseness refers to a difficulty making sounds when trying to speak. Vocal sounds
may be weak, breathy, scratchy, or husky, and the pitch or quality of the voice may
change.
Hoarseness is most often caused by a problem with the vocal cords, which are part of
your voice box (larynx) in the throat. When the vocal cords become inflamed or infected,
they swell. This can cause hoarseness.

42
All the mentioned conditions can result from this kind of trauma and these lead to
hoarseness.
20. D
Laryngopharynx (hypopharynx) can be divided into three sub-sites:
1. piriform sinus
2. postcricoid area
3. posterior pharyngeal wall
21. C
Reports suggest that the decline has mainly involved tonsillectomies performed for
infectious indications, while the number of tonsillectomies performed for obstructive
indications may have actually increased.
22. B
Bezold's abscess, an abscess (a collection of pus surrounded by inflamed tissue)
behind the sternocleidomastoid muscle in the neck and is a complication of mastoiditis.
23. B
It lies between the aryepiglottic fold medially and the wall of the pharynx laterally.
24. D
Vocal cord paralysis is the second most common congenital laryngeal abnormality.
This must be differentiated from the commonest congenital laryngeal abnormality,
laryngomalacia. Stridor is the predominant presenting symptom in both of these
conditions. Other symptoms of vocal cord paralysis in children include obstruction, weak
cry, dysphagia, and aspiration. The diagnosis can usually be made by flexible endoscopy
at the bedside.
25 C
Onset of symptoms of respiratory diphtheria typically follows an incubation period of
2-5 days (range, 1-10 d). Symptoms initially are general and nonspecific, often
resembling a typical viral upper respiratory infection (URI). Respiratory involvement
typically begins with sore throat and mild pharyngeal inflammation. Development of a
localized or coalescing pseudomembrane can occur in any portion of the respiratory tract.
The pseudomembrane is characterized by the formation of a dense, gray debris layer
composed of a mixture of dead cells, fibrin, RBCs, WBCs, and organisms.
Removal of the membrane reveals a bleeding, edematous mucosa. The distribution of
the membrane varies from local (eg, tonsillar, pharyngeal) to widely covering the entire
tracheobronchial tree. The membrane is intensely infectious, and droplet and contact
precautions must be followed when examining or caring for infected patients. A
combination of cervical adenopathy and swollen mucosa imparts a "bull's neck"
appearance to many of the infected patients; this is shown in the image below. The most
frequent cause of death is airway obstruction or suffocation following aspiration of the
pseudomembrane.

43
26. C
 The abscess lies in the potential space between the Buccopharyngeal &
prevertebral fasciae.
 The acute abscess is caused by suppuration in the retropharyngeal lymph nodes,
which become infected from the nasopharynx & oropharynx. The commonest
organism is streptococcus pneumoniae.
27. B
 Nx lymph nodes cannot be assessed
 N0 no regional nodes involved
 N1 single ipsilateral lymph node < 3 cm in diameter
 N2a single ipsilateral lymph node between 3 and 6 cm in diameter
 N2b multiple ipsilateral lymph nodes, none larger than 6 cm diameter
 N2c bilateral or contralateral lymph nodes, none larger than 6 cm diameter
 N3 lymph node larger than 6 cm in diameter
28. E
The subglottis is defined as the area of the larynx housed by the cricoid cartilage that
extends from 5 mm beneath the true vocal cords to the inferior aspect of the cricoid ring.
A reactive inflammatory response causes subglottic edema. Narrowing of the airway can
be life threatening in infants and young children because of their small airway and lax
tissues.
29. B
Tumour staging
Site Stage TNM Classification
Tumour Tx not defined
T0 no primary tumour or carcinoma in situ
T1 One subsite & less than 2 cm diameter in greatest dimention.
T2 More than one subsite & tumour 2 to 4 cm diameter in greatest
dimention. Without fixation of hemilarynx
T3 tumour over 4 cm diameter With fixation of hemilarynx
44
T4 tumour invades surrounding structures
Nodes Nx lymph nodes cannot be assessed
N0 no regional nodes involved
N1 single ipsilateral lymph node < 3 cm in diameter
N2a single ipsilateral lymph node between 3 and 6 cm in diameter
N2b multiple ipsilateral lymph nodes, none larger than 6 cm diameter
N2c bilateral or contralateral lymph nodes, none larger than 6 cm diameter
N3 lymph node larger than 6 cm in diameter
Metastasis Mx not assessed
M0 no distant metastases
M1 distant metastasis present

30. B
It is the disease of young adults and it can be confirmed by monospot test called paul
bunel test. The other conditions usually show more signs and symptoms which are
evident at examination.
31. C
The most common complication of tonsillectomy is Haemorrhage.
32. D
Articulation structures such as palate, tongue, …etc are responsible for generation of
speech.
33. E
Stridor is a high-pitched musical breath sound resulting from turbulent air flow in the
larynx or lower in the bronchial tree. Stridor is a physical sign which is caused by a
narrowed or obstructed airway. It can be inspiratory, expiratory or biphasic, although it is
usually heard during inspiration. Inspiratory stridor often occurs in children with "croup."
It may be indicative of serious airway obstruction from severe conditions such as
epiglottitis, a foreign body lodged in the airway, or a laryngeal tumor. Stridor should
always command attention to establish its cause. Visualization of the airway by medical
experts equipped to control the airway may be needed.
34. D
In order of frequency of occurrence of malignant tumours of hypopharynx:
1. Piriform sinus: 60%
2. Post cricoid area: 30%
3. Postertior pharyngeal wall: 10%
35. E
 Cervical lymphadenopathy (disease or swelling of the lymph nodes in the
neck) is the initial presentation in many patients, and the diagnosis of NPC is
often made by lymph node biopsy.
 Symptoms of local invasion (Trotter‫ۥ‬s triad)
1. Conductive deafness: due to infiltration of the eustachian tube. This may
proceed to secretory otitis media.
2. Elevation and immobility of the homolateral soft palate: due to direct
infil¬tration.

45
3. Pain in the side of the head: due to involvement of the Vth cranial nerve, from
infiltration via the foramen lacerum. Pain may also be felt in the ear, upper or
lower jaws, and the tongue.

 Radiotherapy is the method of choice, because surgical removal of the


primary growth is rarely possible; metastases are often present when the
patient is first seen; and the tumours are usually anaplastic and highly
radiosensitive.
 Nasopharyngeal carcinoma (NPC) is caused by a combination of factors:
viral, environmental influences, and heredity. The viral influence is associated
with infection with Epstein-Barr virus (EBV).
36. C
 Trismus, or lockjaw, refers to reduced opening of the jaws caused by spasm of
the muscles of mastication, or may generally refer to all causes of limited
mouth opening. It is a common problem with a variety of causes, and may
interfere with eating, speech, oral hygiene, and could alter facial appearance.
There is an increased risk of aspiration. Temporary trismus is much more
common than permanent trismus, and may be distressing and painful, and
limit or prevent medical examination or treatments requiring access to the oral
cavity.
 Quinsy because of its proximity to the parapharyngeal musculature there is
associated spasm particularly of the pterygoid muscles.
37. B
Waldeyerʼs ring is a ring of lymphatic tissue formed by the two palatine tonsils, the
nasopharyngeal tonsil (adenoids), the tubal tonsils (around the Eustachian opening), the
lingual tonsil, and intervening lymphoid tissue
38. E
Nasopharyngeal angiofibroma (also called juvenile nasopharyngeal angiofibroma)
is a histologically benign but locally aggressive vascular tumor that grows in the back of
the nasal cavity. It most commonly affects adolescent males. Patients with angiofibroma
usually present with one-sided nasal obstruction and recurrent bleeding.
39. D
Acute epiglottitis: it is a special form of acute laryngitis, in which the inflammatory
changes affect mainly the loosely attached mucosa of the epiglottis. localized edema may
obstruct the air way especially in children, in whom H. influinzae may be the causal
organism. Age incidence isbetween1-6 year old.
40. D
Intravenous antibiotics against H.influinzae in high doses (amoxicillin+ clavulanic
acid) is a must to be started immediately .
41. D
 The mouth extends from the lips to the anterior pillar of the fauces.
 The mouth consists from two parts:-
1. Vestibule.
2. Mouth proper.
 It communicates posteriorly with the oropharynx through the oropharyngeal
isthmus.

46
 The floor is formed by the anterior 2/3 of the tongue
 The sublingual and the submandibular salivary glands open under the tongue
in the floor of the mouth while the duct of the parotid gland opens in to the
vestibule against the second upper molar tooth.
42. A
Extends from base of the skull to the sixth cervical vertebra at lower border of the
cricoid cartilage.
43. D
 The oral vestibule lies between the lips and cheek on one side and the teeth
and alveoli on the other side.
 The mouth proper is the site of drainage of salivary gland which
communicates posteriorly with the oropharynx through the oropharyngeal
isthmus.
44. A
The mylohyoid muscle is one of the depressors of the mandible, with the muscles of
mastication it is supplied by mandibular branch of the trigeminal 5th cranial nerve.

45. C
The vocal cords separate the supraglottis from the subglottis and it is devoid from
lymphatic drainage . This explain the delay in lymphatic metastasis in glottis carcinoma.
46. C
It is an embryological remnant located at the midline of the posterior wall of the
nasopharynx.
47. A
They are lined by respiratory epithelium (pseudostratified ciliated columnar
epithelium with goblet cells).
48. D
Indications for tracheostomy:-
1-relief of upper respiratory obstruction.
a-congenital: bilateral caoanal atresia , laryngeal web, laryngeal cyst ,upper tracheal
stenosis, and tracheo- esophageal anomalies.
b-traumatic: internal (inhalation of steam or irritant fumes, foreign body or swallowing
of corrosive) or external (blow on the larynx , gun shot , or cut throat).
c-infections: acute epiglottitis, acute laryngotracheo bronchitis, diphtheria , Ludwig's
angina , retropharyngeal abscess, or parapharyngeal abscess.
d-tumors: malignant tumors of tongue , upper trachea, , pharynx , larynx , or thyroid
gland.
e- bilateral recurrent laryngeal palsies: after thyroidectomy or bulbar palsy .
f- cord fixation : due to rheumatoid arthritis.
g-other causes of glottic obstruction: like hemophilia , angioneurotic oedema…….
2-protection of lower respiratory tract: bulbar and pseudobulbar palsy, coma (head injury
or drug abuse), myasthenia gravis, polyneuritis, tetanus , cervical cord lesion ,
myasthenia gravis.…

3-assist ventilation: in conditions that cause respiratory insufficiency :-


a-pulmonary disease : chronic bronchitis , emphysema , and postoperative pneumonia .

47
b-severe chest injury: flial chest.
c-neuromuscular incoordination that cause stagnation of bronchial secretion or the need
for artificial or intermittent positive pressure ventilation PPV.
4-elective procedure: in any major procedure in mouth ,pharynx or larynx.
49. A
Aspiration pneumonia is bronchopneumonia that develops due to the entrance of
foreign materials into the bronchial tree, usually oral or gastric contents (including food,
saliva, or nasal secretions). Depending on the acidity of the aspirate, a chemical
pneumonitis can develop, and bacterial pathogens (particularly anaerobic bacteria) may
add to the inflammation.
Aspiration pneumonia is often caused by an incompetent swallowing mechanism, such as
occurs in some forms of neurological disease or injury including multiple sclerosis, CVA
(stroke), Alzheimer's disease or intoxication. An iatrogenic cause is during general
anaesthesia for an operation and patients are therefore instructed to be nil per os (NPO)
(aka Nothing By Mouth) for at least four hours before surgery.
50. D
Post-operative care of tracheostomized patient:-
1. Nursing care: constant attention is essential for the first 24 hours at least.
2. Fixation of the tracheostomy tube.
3. Removal of secretions by suction.
4. Humidification: by wet gauze.
5. Changing the tube.
6. Care of the inflatable cuff.
7. Removal of the tube.

48
Answers of Rhinology questions
1. B
 Nasal polyps are round, smooth, soft, translucent, pale glistening structures. They
are non-tender and move backwards when probed.
 Antro choanal polyp originates from maxillary antrum.
 Ordinary nasal polyp results from prolapsed lining of the both ethmoid sinuses
 An inverted papilloma is a type of tumor in which surface epithelial cells grow
downward into the underlying supportive tissue. It may occur in the nose and/or
sinuses or in the urinary tract (bladder, renal pelvis, ureter, urethra). When it
occurs in the nose or sinuses, it may cause symptoms similar to those caused by
sinusitis, such as nasal congestion. When it occurs in the urinary tract, it may
cause blood in the urine.
 Furuncolosis is an infection of hair follicle of nasal vestibule caused by staph
areus.
 Nasal turbinates are three in number on lateral wall of the nose.
2. D
Allergic Rhinitis is an IgE-mediated hypersensitivity disease of the mucous
membranes of the nasal airways characterized by sneezing, itching, watery nasal
discharge and a sensation of nasal obstruction.
3. B
Atopy represent a predisposition to develop allergic disease. Allergy represent the
clinical expression of atopic disease. The prevalence of atopy is extremely common
affecting up to 1/3 of the general population. It follows that not all atopic individuals
develop symptoms and atopy is genetically inherited.
4. D
 The arterial blood supply of the nose comes from both external internal
carotid artery.
 The venous drainage of the nose has a connection with the sagittal sinus.
 Superficial temporal and maxillary arteries are the terminal branches of the
external carotid artery.
5. C
The inferior turbinate can be seen usually by anterior rhinoscopy, and to less extent is
the middle turbinate. The superior turbinate is usually invisible by anterior rhinoscopy.
Olfactory epithelium lines the upper 1/3 of the nasal septum, the roof of the nose, and the
lateral wall above and including the superior turbinates. It is yellow in color, non-ciliated
columnar epithelium and usually invisible by anterior rhinoscopy.
Nasal endoscopy is a major part in routine ENT examination.
6. D
lymphatic drainage of the nose from the anterior part of the nose to the submandibular
lymph nodes and the superior deep cervical chain. the posterior is drained to the superior
and middle deep cervical chain either directly or after passing through the
retropharyngeal lymph nodes.

49
7. E
 It is an IgE mediated.
 Commonly associated with bronchial asthma.
 Paroxysms of sneezing occur at frequent intervals throughout the day is the
main symptom.
 It is associated with watery nasal discharge.
8. C
Treatment of non-eosinophilic vasomotor rhinitis: Most patients are controlled by
medical means and only minority require surgical treatment.
Medical treatment include anti-cholinergic local drops : topical ipratrobium bromide.
While surgical treatmentinclude vidian neurectomy to control rhinorrhea.
9. C
 It is type 1 allergic hypersensitivity reaction.
 It involves IgE reaction.
 The mast cells have a major role in pathogenesis.
 Eosinoophil count is essential for diagnosis.
 Atopy refers to the tendency to develop an exaggerated IgE Ab response as
reflected by skin prick test in response to one or more of common
aeroallergens. It follows that atopy represent a predisposition to develop
allergic disease.
Allergy represent the clinical expression of atopic disease.
10. C
The external nose:- It is pyramidal in shape base cranially and tip caudally ,its shape is
maintained by skeletal framework which is composed of:-
1-bony constituent 2-cartilagenous constituents.
The bony constituents supports the upper 1\3 of the nose and mainly formed by the nasal
bones. the cartilaginous constituents support the lower 2\3 of the external nose and
formed by:-1-upper lateral cartilage 2-lower lateral cartilage 3-quadrilatewral cartilage of
the nasal septum.
11. E
Kiesselbach's plexus, which lies in Kiesselbach's area, Kiesselbach's triangle, or
Little's area, is a region in the anteroinferior part of the nasal septum where four arteries
anastomose to form a vascular plexus of that name. The arteries are:
1. Anterior ethmoidal artery (from the ophthalmic artery)
2. Sphenopalatine artery (terminal branch of the maxillary artery)
3. Greater palatine artery (from the maxillary artery)
4. Septal branch of the superior labial artery (from the facial artery)
12. B
13. D
The nasal vestibule is the most anterior part of the nasal cavity. It is enclosed by the
cartilages of nose and lined by the same epithelium of the skin (stratified squamous,
keratinized). The other part of the nasal cavity, which is lined by the respiratory
epithelium, is called nasal cavity proper. Inside the vestibule are small hairs called
vibrissae, which filter dust and other matter that are breathed in.
The vestibule can be seen by just elevation of the tip of the nose.

50
14. D
The vomer is one of the unpaired facial bones of the skull. It is located in the
midsagittal line, and articulates with the sphenoid, the ethmoid, the left and right palatine
bones, and the left and right maxillary bones. The vomer forms the posterior part of the
nasal septum, with the anterior part formed by the ethmoid.
15. A
The skin lines the nasal vestibule only.
16. C
It is a non-infective, non-allergic condition. It is divide into two groups:-
1. Eosinophilic (obstructive) rhinitis.
2. Non-eosinophilic (non-obstructive) rhinitis.
The etiology is still obscure, although it seems that eosinophilic intrinsic rhinitis is
associated with both autonomic imbalance and local inflammation, and the main
symptom is nasal obstruction. On the other hand non-eosinophilic intrinsic rhinitis is
associated with severe rhinorrhea as its main symptom and autonomic imbalance
(parasympathetic over stimulation) as its main pathological feature.
17. D
Paranasal sinuses are a group of four paired air-filled spaces that surround the nasal
cavity (maxillary sinuses), above the eyes (frontal sinuses), between the eyes (ethmoidal
sinuses), and behind the ethmoids (sphenoidal sinuses). The sinuses are named for the
facial bones in which they are located.
The biological role of the sinuses is debated, but a number of possible functions have
been proposed:
1. Decreasing the relative weight of the front of the skull, and especially the bones
of the face.
2. Increasing resonance of the voice.
3. Providing a buffer against blows to the face.
4. Insulating sensitive structures like dental roots and eyes from rapid temperature
fluctuations in the nasal cavity.
5. Humidifying and heating of inhaled air because of slow air turnover in this region.
6. Regulation of intranasal and serum gas pressures
7. Immunological defense
Despite these various proposals, the paranasal sinuses may not serve any biological
function at all.
18. E
 A boil, also called a furuncle, is a deep folliculitis, infection of the hair follicle.
It is most commonly caused by infection by the bacterium Staphylococcus
aureus, resulting in a painful swollen area on the skin caused by an
accumulation of pus and dead tissue.
 Treatment: broad spectrum antibiotics ,with local ointment.
 Complications: cellulites of upper lip and cavernous sinus thrombosis due to
retrograde venous drainage therefore broad spectrum antibiotics should be
given as soon as possible.

51
19. A
 The nose is the most frequently injured facial structure. In the setting of
trauma to the anterior nasal septum, hematoma formation may occur.
 The anterior portion of the nasal septum is composed of a thin cartilaginous
plate with a closely adherent perichondrium and mucosa. Sub-mucosal blood
vessels are torn as buckling forces pull the perichondrium from the cartilage.
 It is mostly bilateral.
 It is mainly treated by incision and drainage.
20. E
Small cases of septal deviation is treated surgically by:
1. Sub-mucosal resection.
2. Septoplasty.
3. Septorhinoplasty.
21. B
A rhinolith is a calculus present in the nasal cavity. The word is derived from the roots
rhino- and -lith, literally meaning "nose stone". It is an uncommon medical phenomenon,
not to be confused with dried nasal mucus. A rhinolith usually forms around the nucleus
of a small exogenous foreign body, blood clot or secretion by slow deposition of calcium
and magnesium salts. Over a period of time, they grow into large irregular masses that fill
the nasal cavity. They may cause pressure necrosis of the nasal septum or lateral wall of
nose. Rhinoliths can cause nasal obstruction, epistaxis, headache, sinusitis and epiphora.
Clinical features
Rhinoliths present as unilateral nasal obstruction. Foul-smelling, blood-stained discharge
is often present. Epistaxis and pain may occur due to the ulceration of surrounding
mucosa.
22. C
A nasal fracture, commonly referred to as a broken nose, is a fracture of one of the bones
of the nose. Because of the protrusion of the nose from the face and the fragility of the
bones of the nose, a broken nose is one of the most common facial injuries, comprising
almost 40% of all facial injuries.
Causes
Nasal fractures are caused by physical trauma to the face. Common sources of nasal
fractures include sports injuries, fighting, falls, and car accidents in the younger age
groups, and falls from syncope or impaired balance in the elderly.
Symptoms
Symptoms of a broken nose include bruising, swelling, tenderness, pain, deformity,
and/or bleeding of the nose and nasal region of the face. The patient may have difficulty
breathing, or excessive nosebleeds (if the nasal mucosa are damaged).
Diagnosis
Nasal fractures are usually identified visually and through physical examination. A
priority is to distinguish simple fractures limited to the nasal bones (Type 1) from
fractures that also involve other facial bones and/or the nasal septum (Types 2 and 3). In
simple Type 1 fractures X-Rays supply surprisingly little information beyond clinical
examination. However, diagnosis may be confirmed with X-rays or CT scans, and these
are required if other facial injuries are suspected. Although treatment of an
uncomplicated fracture of nasal bones is not urgent, referral for specific treatment in five

52
to seven days usually suffices, an associated injury, nasal septal hematoma, occurs in
about 5% of cases and does require urgent treatment and should be looked for during the
assessment of nasal injuries.
Treatment
Minor nasal fractures may be allowed to heal on their own provided there is not
significant cosmetic deformity. Ice and pain medication may be prescribed to ease
discomfort during the healing process. For nasal fractures where the nose has been
deformed, manual alignment may be attempted, usually with good results. Injuries
involving other structures (Types 2 and 3) must be recognized and treated surgically.
23. A
 Decongestants seem to work the best to resolve pain, but only temporarily.
 It is synonymous to anterior ethmoid neuralgia.
 Pain localized to a single area on the face on just ONE side.
 Due to compression of deviated septum and middle or superior turbinate.
 Pain described as sharp or shooting; less commonly as pressure (like someone
driving a high-heel into the face).
24. A
Viral (10-15%)
 Rhinovirus most common
 Influenza
 Parainfluenza
 Adenovirus
Bacterial (85-90%)
 S. Pneumonia
 H. influenzae
 Moraxella cattarhalis
25. B
 Headache/facial pain or pressure of a dull, constant, or aching sort over the
affected sinuses is common with both acute and chronic stages of sinusitis.
This pain is typically localized to the involved sinus and may worsen when
the affected person bends over or when lying down. Pain often starts on one
side of the head and progresses to both sides.
 The cases of open sinusitis are free of pain.
 Otalgia might be due referred pain or acute otitis media.
 Antibiotic and decongestant with pain killers share in relieving the pain.
26. C
Adenoids causing obstruction of both choana. Allergic rhinitis causing alternating
bilateral nasal obstruction. Ethmoidal polyps are multiple bilateral polyps arising from
the ethmoidal sinuses. Despite wide roomy nose in atrophic rhinitis it causes bilateral
nasal obstruction because of crustation and absence of the feeling of air current.

27. D
Glomerulonephritis may be a complication of B-haemolytic streptococcus tonsillitis.

53
28. E
 The vast majority of cases of sinusitis are caused by viruses and will therefore
resolve without antibiotics.
 Nasal endoscopy involves inserting a flexible fiber-optic tube with a light and
camera at its tip into the nose to examine the nasal passages and sinuses. This is
generally a completely painless (although uncomfortable) procedure which takes
between five to ten minutes to complete.
 CT scan is recommended for chronic sinusitis to delineated the extent of disease
and the anatomy of sinuses.
 Imaging by either X-ray, CT or MRI is generally not recommended in acute
sinusitis unless complications develop because most cases resolves with simple
measures.
 Minor cases of acute sinusitis need surgery.
29. C
 Spherical foreign body removed by Jobson horn probe.
 The signs and symptoms of a nasal foreign body are
1. Pain in the child’s nose or sinuses
2. Trouble breathing through his nose
3. Bad breath
4. Bloody nose
5. A headache, itching, or sneezing
 Button batteries and paired disc magnets can cause serious damage to nasal
structures and merit urgent removal.
 Intranasal foreign bodies (FBs) occur most commonly in young children.
 FBs are more frequently located on the right side due to the predominance of
right-handed children.
30. C
Local factors include: -
1. Any nasal obstruction, septal deviation, polyps, turbinate hypertrophy.
2. Neibouring infection.
3. Obstruction of sinus ostia: by vasomotor rhinitis or allergic rhinitis.
4. Acute on chronic sinusitis.
31. B
32. B
Etiology of epistaxis:- mostly idiopathic.
A. Local:
1. spontaneuous bleeding from little’s area ,site of anastomosis of four arteries from
the internal and external carotid arteries . it bleeds on minor trauma especially in
children there is a prominent plexus of blood vessels.
2. trumatic with a brasion on nasal mucosa following a blow on the nose , fracture
nasal bone , fracture base of skull or fracture paranasal sinuses.
3. inflammatory: in acute or chronic rhinitis , in bacterial or viral infections.
4. neoplastic: benign or malignant tumors in the nose , paranasal sinuses or
nasopharynx. e.g. haemangioma of the septum ,angiofibroma of the nasopharynx.
B. General:

54
1. raised arterial pressure: hypertension due to atherosclerosis or renal disease.
2. raised venous pressure : as in cardiac or pulmonary disease e.g. mitral stenosis or
cor pulmonale.
3. disease of blood or blood vessels :e.g. leukemia , hemophilia , purpura ,
deficiency of vitamine C , K , or Ca++, chrismas disease , osler’s disease,…
33. E
 The diagnosis as well as the degree of impairment can now be tested much
more efficiently and effectively than ever before thanks to "smell testing kits"
that have been made available as well as screening tests which use materials
that most clinics would readily have.
 Many patients may experience unilateral anosmia, often as a result of minor
head trauma.
 It is often described as loss of taste but the mechanism of their association is
not understood.
 Esthesioneuroblastoma is an exceedingly rare cancerous tumor that originates
in or near the olfactory nerve. Symptoms are anosmia (loss of sense of smell)
often accompanied by chronic sinusitis.
 Anosmia may be temporary but traumatic anosmia (Head trauma, damage to
the ethmoid bone) can be permanent.
34. D
 Otitis media is a recognized complication of common cold.
 Quinsy is a recognized complication of acute tonsillitis.
 Facial palsy may be a complication of otitis media.
 Common cold is the most common cause of maxillary sinusitis.
 An inverted papilloma is a type of tumor in which surface epithelial cells grow
downward into the underlying supportive tissue. It may occur in the nose
and/or sinuses or in the urinary tract (bladder, renal pelvis, ureter, urethra).
When it occurs in the nose or sinuses, it may cause symptoms similar to those
caused by sinusitis, such as nasal congestion. When it occurs in the urinary
tract, it may cause blood in the urine.
35. E
Headache doesn’t occur in uncomplicated cases of the boil of nasal vestibule.
36. C
Mast cell stabilizers are cromone medications used to prevent or control certain
allergic disorders. They block a calcium channel essential for mast cell degranulation,
stabilizing the cell and thereby preventing the release of histamine[1] and related
mediators. One suspected pharmacodynamic mechanism is the blocking of IgE-regulated
calcium channels. Without intracellular calcium, the histamine vesicles cannot fuse to the
cell membrane and degranulate.
As inhalers they are used to treat asthma, as nasal sprays to treat hay fever (allergic
rhinitis) and as eye drops for allergic conjunctivitis.
37. D
The antrochoanal polyp is uncommon. It is typically unilateral (may be bilateral)
and commences as oedematous lining from the maxillary sinus. This lining prolapses
through the ostium into nasal cavity and enlarges towards the posterior choana and
nasopharynx.

55
38. B
Immunotherapy (hypo-sensitization):- this mode of treatment involves the injection
of small amount of the antigen to mop up the allergen specific Ig in the patient. The
main problem is the possibility of anaphylaxis .
39. A
Sphenoid sinus open in sphenoethoidal recess.
40. E
we have four stages:-
1. ischemic phase: cause burning sensation in the nasopharynx and soft palate, cold
sensation, and sneezing.
2. Hyperemic stage:-watery rhinorrhea with mild fever.
3. Stage of secondary infection: watery rhinorrhea but thicker and yellowish in
color.
4. Stage of resolution: about 5 days and if the patient has low resistant it may
become 10 days.
41. C
Treatment include:
4. General: absolute bed rest, plenty of fluid, analgesia, anti-histamine,
decongestant, antibiotics only in secondary bacterial infections.
5. Local: steam inhalation, local vasoconstrictive, nasal sprays and drops.
42. E
Dullness on trans-illumination is a sign (not a symptom) of acute maxillary sinusitis.
43. D
All X-ray views are not as accurate as CT- scan view.
44. B
Coronal CT scan view is the best to show the details of maxillary sinus.
45. A
Seasonal rhinitis:- the first symptom is usually sneezing, in severe cases paroxysms
of sneezing occur at frequent intervals throughout the day. Excessive fluid and
mucous secretion (rhinorrhea), nasal obstruction, itching of nose, eye and palate are
other symptoms. Conjunctivitis is common associate.
46. D
Avoidance of the allergens is the optimum treatment. Unfortunately, this rarely
possible totally.
47. D
The commonest cause is allergy to house dust mites. Other major perennial
allergens include domestic pets (cats, dogs, rabbits, hamsters, and horses).
48. D
49. E
The patient, commonly a young adult, complains of unilateral nasal obstruction which
is worse on expiration owing to the ball valve-like effect of the polyp in the posterior
choana.

56
50. C
Etiology of acute maxillary sinusitis: nasal origin 90% and dental origin10%.
1. Acute infective rhinitis.
2. Swimming and diving in contaminated water.
3. Fracture of maxillary sinus.
4. Barotrauma.

57

Você também pode gostar